You are on page 1of 9

Gleim CMA Test Prep: Part 2: Financial Decision Making

Answer Explanations
(19 questions)

[1]

Answer (A) is incorrect because The amount of $6 million is the increase in cash, not the interest earned on that additional
cash.

Answer (B) is correct. If cash outflows are $3 million per day, holding cash 2 extra days means that average balances
should increase by $6 million. At a 10% interest rate, the additional $6 million would generate interest revenue of $600,000
per year. Thus, if the system can be acquired for $600,000 or less, it would be beneficial to do so.

Answer (C) is incorrect because The daily payments should be multiplied by two, not divided by two.

Answer (D) is incorrect because The amount of $3 million is the amount of daily payments, not the savings.

[2]

Answer (A) is correct. Checks are currently tied up for 11 days (6 for mailing, 3 for processing, and 2 for clearing). If that
period were reduced to 3 days, DLF’s cash balance would increase by $1,200,000 ($150,000 per day × 8 days).

Answer (B) is incorrect because The lockbox system will result in an additional 8 days of savings, not 3.

Answer (C) is incorrect because The decrease is 8 days, not 5.

Answer (D) is incorrect because The amount of $600,000 represents only a 4-day savings.

[3]

Answer (A) is incorrect because This figure results from subtracting the interest earned from the cost.

Answer (B) is incorrect because This figure results from subtracting the service charge for only a single month.

Answer (C) is incorrect because This figure results from failing to subtract the $36,000 cost of the service.

Answer (D) is correct. The additional annual income (loss) from using the bank’s proposed service is the excess (deficit) of
interest earned on the early deposits over (under) the cost of the service. If the plan is adopted, the firm’s average cash
balance will increase by $600,000 ($300,000 × 2 days).

Benefit (loss) = Interest earned – Cost


= ($600,000 × 11%) – ($3,000 × 12 months)
= $66,000 – $36,000
= $30,000

Copyright 2008 Gleim Publications, Inc. Page 1


Printed for M Awais Naseem
Gleim CMA Test Prep: Part 2: Financial Decision Making
Answer Explanations
(19 questions)

[4]

Answer (A) is correct. The annual benefit from using the lockbox system is the excess of interest earned on the early
deposits over the cost of the service. If the plan is adopted, the firm’s average cash balance will increase by $2,000,000
($20,000 average payment × 50 per day × 2 days). The annual variable cost will be $9,125 ($.50 per payment × 50 per day ×
365 days).

Benefit (loss) = Interest earned – Cost


= ($2,000,000 × 6%) – ($50,000 + $9,125)
= $120,000 – $59,125
= $60,875

Answer (B) is incorrect because The annual savings without regard to costs is $120,000.

Answer (C) is incorrect because The annual fixed fee is $50,000.

Answer (D) is incorrect because The annual lockbox cost is $59,125.

[5]

Answer (A) is incorrect because This figure results from improperly adding, rather than subtracting, the cost of the lockbox.

Answer (B) is incorrect because This figure results from failing to subtract the cost of the lockbox.

Answer (C) is correct. The annual benefit (loss) from using the lockbox system is the excess (deficit) of interest earned on
the early deposits over (under) the cost of the service. If the plan is adopted, Foster’s average cash balance will increase by
$750,000 [$90,000,000 × (3 days ÷ 360 days)].

Benefit (loss) = Interest earned – Cost


= ($750,000 × 8%) – $80,000
= $60,000 – $80,000
= $(20,000)

Answer (D) is incorrect because This figure is the interest revenue that would be earned on the early deposits.

Copyright 2008 Gleim Publications, Inc. Page 2


Printed for M Awais Naseem
Gleim CMA Test Prep: Part 2: Financial Decision Making
Answer Explanations
(19 questions)

[6]

Answer (A) is incorrect because The relevant cost of the minimum premium account deposit is based on the $2,000
incremental deposit, not the full $2,500.

Answer (B) is incorrect because The savings on the premium account is $16.

Answer (C) is incorrect because The savings on the premium account is $16.

Answer (D) is correct. Kemple can compare the costs of the two alternatives as follows:

Standard account = Variable cost + Fixed cost


= [(80 checks × $.10) + $10] × 12 months
= $18 × 12 months
= $216 per year

Premium account = Variable cost + Fixed cost


= [$0 + ($2,500 reqd. – $500 projected)] × 10%
= $2,000 × 10%
= $200 per year

Thus, the premium account should be selected because it is cheaper by $16 per year.

[7]

Answer (A) is incorrect because The bank float is also a cost-effective method.

Answer (B) is incorrect because The lockbox system is also a cost-effective method.

Answer (C) is correct. The total cost of each of the four methods that Rolling Stone is considering can be calculated as
follows:

Lockbox: $25 per-bank fee × 170 banks = $ 4,250


Drafts: $2 per-draft fee × 4,000 drafts = $ 8,000
Bank Float: $1,000,000 in checks written × 2% fee = $20,000
Electronic Transfer: $18 per-transfer fee × 700 items = $12,600

These costs are subtracted from the interest that could be earned under each method to arrive at the relevant gain or loss:

Lockbox: $5,240 – $4,250 = $ 990


Drafts: $6,500 – $8,000 = $(1,500)
Bank Float: $22,000 – $20,000 = $ 2,000
Electronic Transfer: $14,000 – $12,600 = $ 1,400

The lockbox system, the bank float, and the electronic transfer are cost-effective.

Answer (D) is incorrect because Drafts are not a cost-effective method.

Copyright 2008 Gleim Publications, Inc. Page 3


Printed for M Awais Naseem
Gleim CMA Test Prep: Part 2: Financial Decision Making
Answer Explanations
(19 questions)

[8]

Answer (A) is correct. To break even, the interest that JKL can earn on the early deposits must at least equal the excess of
the wire transfer fee over the cost of the DTC.

Transfer amount × 9% × (2 days ÷ 360 days) = $12 WT – $1.50 DTC


Transfer amount × .05% = $10.50
Transfer amount = $21,000

Answer (B) is incorrect because The amount of $24,000 results from using the full cost of the wire transfer rather than the
incremental cost.

Answer (C) is incorrect because The amount of $27,000 results from adding, rather than subtracting, the cost of the
depository transfer check to the cost of the wire transfer.

Answer (D) is incorrect because The amount of $42,000 results from using only 1 day of interest rather than 2.

[9]

Answer (A) is incorrect because The transactions cost of bankers’ acceptances is high. A banker’s acceptance is a unique
credit instrument used to finance both domestic and international “self-liquidating” transactions. It is usually initiated by a
bank’s irrevocable letter of credit on behalf of the bank’s customer, on which the company doing business with the bank’s
customer draws a time draft. The company discounts the time draft with the company’s local bank and receives immediate
payment. The local bank forwards the time draft to the bank customer for payment.

Answer (B) is correct. A small firm with surplus cash should invest for the highest return and lowest risk. The ability to
convert the investment into cash without a loss of principal is also important. Money market mutual funds invest in money
market certificates such as treasury bills, negotiable CDs, and commercial paper. Because of diversification, these mutual
funds are superior to any single instrument.

Answer (C) is incorrect because A small firm may not have enough surplus cash to invest in commercial paper, which
usually consists of secured or unsecured promissory notes of large corporations.

Answer (D) is incorrect because An increase in interest rates could cause a substantial loss in principal.

Copyright 2008 Gleim Publications, Inc. Page 4


Printed for M Awais Naseem
Gleim CMA Test Prep: Part 2: Financial Decision Making
Answer Explanations
(19 questions)

[10]

Answer (A) is incorrect because Option 3 has a purchase price of $97.33.

Answer (B) is correct. To determine the amount of interest the lender will earn, the 3.5% discount rate is multiplied by the
face amount of the Treasury bill. The interest on this Treasury bill is $3.50 ($100 × 3.5% × 1 year). Thus, the purchase price
is $96.50 ($100 – $3.5).

Answer (C) is incorrect because Option 4 has a purchase price of $97.00.

Answer (D) is incorrect because Option 1 has a purchase price of $97.00.

[11]

Answer (A) is correct. Borrowing at the bank has a rate of 12%. The commercial paper has an annualized rate of 13.6%
{[($9.1 million – $8.8 million) ÷ $8.8 million] × 4 quarters}. Not taking the trade discount has an annualized rate of
12.245% {(2% ÷ 98%) × [360 days ÷ (90 – 30 days)]}.

Answer (B) is incorrect because The differences in cost are significant given the amounts involved.

Answer (C) is incorrect because The annual cost of the trade discount is 12.245% (2/98 × 6 times per year).

Answer (D) is incorrect because The annualized cost of the commercial paper is 13.6%.

Copyright 2008 Gleim Publications, Inc. Page 5


Printed for M Awais Naseem
Gleim CMA Test Prep: Part 2: Financial Decision Making
Answer Explanations
(19 questions)

[12]

Answer (A) is incorrect because The annual yield on the 360-day investment is only 10%.

Answer (B) is incorrect because The annual yield on the 270-day investment is only 6.7%.

Answer (C) is incorrect because The annual yield on the 180-day investment is only 12%.

Answer (D) is correct. The first step is to calculate the nominal return that each investment will provide:

Amount Nominal
Term Invested Discount Return
90 days $ 80,000 × 5% = $4,000
180 days 75,000 × 6% = 4,500
270 days 100,000 × 5% = 5,000
360 days 60,000 × 10% = 6,000

The next step is to restate the nominal returns on an annualized basis:

Nominal Fraction Annualized


Term Return of Year Return
90 days $4,000 ÷ (90 ÷ 360) = $16,000
180 days 4,500 ÷ (180 ÷ 360) = 9,000
270 days 5,000 ÷ (270 ÷ 360) = 6,667
360 days 6,000 ÷ (360 ÷ 360) = 6,000

The final step is to calculate the annual yield of each investment.

Annualized Amount Annual


Term Return Invested Yield
90 days $16,000 ÷ $ 80,000 = 20.0%
180 days 9,000 ÷ 75,000 = 12.0%
270 days 6,667 ÷ 100,000 = 6.7%
360 days 6,000 ÷ 60,000 = 10.0%

Copyright 2008 Gleim Publications, Inc. Page 6


Printed for M Awais Naseem
Gleim CMA Test Prep: Part 2: Financial Decision Making
Answer Explanations
(19 questions)

[13]

Answer (A) is incorrect because The amount of $13,778 is the average daily sales under the new policy.

Answer (B) is incorrect because The amount of $98,890 is the amount of the additional receivables.

Answer (C) is correct. The projected average balance in receivables under the old policy was $383,333 [$4,600,000 × (30
days ÷ 360 days)]. Under the new policy, the average balance will be $482,222 [$4,960,000 × (35 days ÷ 360 days)]. Hence,
the average balance is $98,889 higher under the new policy ($482,222 – $383,333). The pre-tax cost of carrying the
additional investment in receivables can be calculated as follows:

Increased investment in receivables -- gross $98,889


Times: variable cost ratio × 50%
Increased investment in receivables -- net $49,444
Times: opportunity cost of funds × 11%
Incremental cost of new credit plan $ 5,439

Answer (D) is incorrect because The amount of $10,878 results from failing to adjust for the proportion of incremental costs
included in the additional receivables.

[14]

Answer (A) is incorrect because The amount of $30,000 results from using a 5-day lengthening rather than a 6-day.

Answer (B) is correct. If the change is adopted, Best’s average balance in receivables will increase by $450,000
{$27,000,000 × [(34 days – 28 days) ÷ 360 days]}. The minimum savings that Best must experience to justify the change is
therefore $36,000 ($450,000 × 8%).

Answer (C) is incorrect because The amount of $180,000 results from using 30 days rather than 6 days.

Answer (D) is incorrect because The amount of $360,000 results from simply dividing annual sales by 6 days rather than
weighting it for an entire year.

Copyright 2008 Gleim Publications, Inc. Page 7


Printed for M Awais Naseem
Gleim CMA Test Prep: Part 2: Financial Decision Making
Answer Explanations
(19 questions)

[15]

Answer (A) is incorrect because If the cost of stockouts declines, the incentive to carry large safety stocks is reduced.

Answer (B) is incorrect because A decline in the variability of usage makes it easier to plan orders, and safety stocks will be
less necessary.

Answer (C) is correct. A company maintains safety stocks to protect itself against the losses caused by stockouts. These can
take the form of lost sales or lost production time. Safety stock is necessary because of the variability in lead time and usage
rates. As the variability in lead time increases, a company will tend to carry larger safety stocks.

Answer (D) is incorrect because An increase in inventory carrying costs makes it less economical to carry safety stocks.

[16]

Answer (A) is incorrect because The amount of $6,600 results from using the highest rather than the average inventory and
from excluding the discounts lost.

Answer (B) is incorrect because The amount of $1,600 equals the ordering costs.

Answer (C) is correct. Valley’s annual ordering costs are $1,600 ($200 per order × 8 orders per year), and the annual
carrying costs are $2,500 ($100 per pound × 25 pounds average inventory). Gross annual product purchase cost is $200,000
($500 per pound × 400 pounds annual usage). Because the differential discount lost is 4% (6% – 2%), annual discounts lost
equal $8,000 ($200,000 × 4%). If the purchasing manager places 8 orders of 50 pounds each, Valley’s total cost can be
calculated as follows:

Annual ordering costs $ 1,600


Annual carrying costs 2,500
Annual discounts lost 8,000
Total cost $12,100

Answer (D) is incorrect because The amount of $4,100 results from excluding the discounts lost.

[17]

Answer (A) is incorrect because One of the purposes of holding safety stock is to protect against outages caused by shipping
delays from the supplier.

Answer (B) is incorrect because One of the purposes of holding safety stock is as a hedge against fluctuations in demand.

Answer (C) is incorrect because One of the purposes of holding safety stock is to be able to fill all customer orders.

Answer (D) is correct. Safety stock cannot protect against the entry of new competitors.

Copyright 2008 Gleim Publications, Inc. Page 8


Printed for M Awais Naseem
Gleim CMA Test Prep: Part 2: Financial Decision Making
Answer Explanations
(19 questions)

[18]

Answer (A) is incorrect because Carrying cost is in the denominator of the EOQ fraction. An increase results in a decrease in
the EOQ.

Answer (B) is incorrect because Decisions about the level of safety stock do not affect the EOQ.

Answer (C) is correct. Fixed cost per order is in the numerator of the EOQ fraction. An increase results in an increase in the
EOQ.

Answer (D) is incorrect because The purchase price has no effect on the EOQ.

[19]

Answer (A) is correct.

EOQ = Sq. root [(2 × fixed order cost × periodic demand) ÷ unit carrying cost]
= Sq. root [(2 × $10 × 1,000 units) ÷ ($10 × 20%)]
= Sq. root ($20,000 ÷ $2)
= Sq. root (10,000)
= 100 units

Answer (B) is incorrect because The amount of 45 units results in a higher total cost (carrying costs plus ordering costs) than
quantities of 100.

Answer (C) is incorrect because The amount of 1,000 units results in a higher total cost (carrying costs plus ordering costs)
than quantities of 100.

Answer (D) is incorrect because The amount of 141 units results in a higher total cost (carrying costs plus ordering costs)
than quantities of 100.

Copyright 2008 Gleim Publications, Inc. Page 9


Printed for M Awais Naseem

You might also like